Last visit was: 25 Apr 2024, 07:17 It is currently 25 Apr 2024, 07:17

Close
GMAT Club Daily Prep
Thank you for using the timer - this advanced tool can estimate your performance and suggest more practice questions. We have subscribed you to Daily Prep Questions via email.

Customized
for You

we will pick new questions that match your level based on your Timer History

Track
Your Progress

every week, we’ll send you an estimated GMAT score based on your performance

Practice
Pays

we will pick new questions that match your level based on your Timer History
Not interested in getting valuable practice questions and articles delivered to your email? No problem, unsubscribe here.
Close
Request Expert Reply
Confirm Cancel
SORT BY:
Date
Tags:
Show Tags
Hide Tags
Senior Manager
Senior Manager
Joined: 02 Oct 2009
Posts: 310
Own Kudos [?]: 3793 [20]
Given Kudos: 412
GMAT 1: 530 Q47 V17
GMAT 2: 710 Q50 V36
WE:Business Development (Consulting)
Send PM
Most Helpful Reply
Math Expert
Joined: 02 Sep 2009
Posts: 92913
Own Kudos [?]: 618949 [5]
Given Kudos: 81595
Send PM
General Discussion
User avatar
Manager
Manager
Joined: 13 Dec 2009
Posts: 78
Own Kudos [?]: 555 [2]
Given Kudos: 10
Send PM
User avatar
Manager
Manager
Joined: 19 Nov 2009
Posts: 163
Own Kudos [?]: 277 [2]
Given Kudos: 44
 Q47  V35
Send PM
Re: Brownlea’s post office must be replaced with a larger one. The present [#permalink]
2
Kudos
The option which states that the construction costs will increase at the city outskirts will weaken the conclusion. Hence, B !
Senior Manager
Senior Manager
Joined: 02 Oct 2009
Posts: 310
Own Kudos [?]: 3793 [4]
Given Kudos: 412
GMAT 1: 530 Q47 V17
GMAT 2: 710 Q50 V36
WE:Business Development (Consulting)
Send PM
Re: Brownlea’s post office must be replaced with a larger one. The present [#permalink]
4
Kudos
Yup Every one is correct the OA is B, But i have a small doubt

if we refer the stem
Quote:
Brownlea’s post office must be replaced with a larger one. The present one cannot be expanded


ok three choice

1)Remain in the old post office building
2)build a new one in the City
3)build a new one in the outskirts

1)Remain in the old post office (is not possible because it really requires a bigger one(stated above)
this says the post office should surely be made a bigger one....(main criteria)....

Quote:
Land near the present location in the center of town is more expensive than land on the outskirts of town. Since the cost of acquiring a site is a significant part of the total construction cost


it clearly states that the land is a significant part of total expenditure.

so building in the City costs 1000Rs
&
building in the outskirts costs 250Rs..

and since its stated acquiring land costs the majority of the expenditure----parking lot cant cost more than 250 (majority more than 50%)

so building a PO(along with parking lot) cant cost more than 500,which is cheaper than building in the city....

and option B says
Quote:
If the new post office is built on the outskirts of town, it will require a parking lot, but if sited near the present post office it will not...


Option b never mentioned the cost Parking lot (it may be too costly or need not be). there is a possibility that building PO(outskirts)+parking lot may be cheaper than building PO in the city alone

Please explain Guys
User avatar
VP
VP
Joined: 06 Sep 2013
Posts: 1345
Own Kudos [?]: 2391 [0]
Given Kudos: 355
Concentration: Finance
Send PM
Re: Brownlea’s post office must be replaced with a larger one. The present [#permalink]
RaviChandra wrote:
Brownlea’s post office must be replaced with a larger one. The present one cannot be expanded. Land near the present location in the center of town is more expensive than land on the outskirts of town. Since the cost of acquiring a site is a significant part of the total construction cost, the post office clearly could be built more cheaply on the outskirts of town.

Which one of the following, if true, most seriously undermines the argument’s stated conclusion?

(A) The new post office will have to be built in accordance with a demanding new citywide building code.
(B) If the new post office is built on the outskirts of town, it will require a parking lot, but if sited near the present post office it will not.
(C) If the new post office is built on the outskirts of town, current city bus routes will have to be expanded to provide access.
(D) If the new post office is built on the outskirts of town, residents will make decreased use of post office boxes, with the result that mail carriers will have to deliver more mail to homes.
(E) If the new post office is built near the center of town, disruptions to city traffic would have to be minimized by taking such steps as doing some construction work in stages at night and on weekends.


I picked D as well...Would anyone clarify what's the OA please?
Cheers!
J :)

Kudos Rain!
User avatar
Retired Moderator
Joined: 16 Jun 2012
Posts: 871
Own Kudos [?]: 8554 [1]
Given Kudos: 123
Location: United States
Send PM
Re: Brownlea’s post office must be replaced with a larger one. The present [#permalink]
1
Kudos
jlgdr wrote:
RaviChandra wrote:
Brownlea’s post office must be replaced with a larger one. The present one cannot be expanded. Land near the present location in the center of town is more expensive than land on the outskirts of town. Since the cost of acquiring a site is a significant part of the total construction cost, the post office clearly could be built more cheaply on the outskirts of town.

Which one of the following, if true, most seriously undermines the argument’s stated conclusion?

(A) The new post office will have to be built in accordance with a demanding new citywide building code.
(B) If the new post office is built on the outskirts of town, it will require a parking lot, but if sited near the present post office it will not.
(C) If the new post office is built on the outskirts of town, current city bus routes will have to be expanded to provide access.
(D) If the new post office is built on the outskirts of town, residents will make decreased use of post office boxes, with the result that mail carriers will have to deliver more mail to homes.
(E) If the new post office is built near the center of town, disruptions to city traffic would have to be minimized by taking such steps as doing some construction work in stages at night and on weekends.


I picked D as well...Would anyone clarify what's the OA please?
Cheers!
J :)

Kudos Rain!


Hello J.

I'm glad to help.

Probably, you missed the conclusion a bit. So D is quite attractive answer, but D is wrong. OA is B.

ANALYZE THE STIMULUS:

Fact: Brownlea’s post office must be replaced with a larger one. The present one cannot be expanded. Fact: Land near the present location in the center of town is more expensive than land on the outskirts of town.
Fact: Since the cost of acquiring a site is a significant part of the total construction cost,
Conclusion: the post office clearly could be built more cheaply on the outskirts of town.

Determining conclusion correctly is KEY. The conclusion only focuses on CONSTRUCTION COST. The conclusion does NOT address future revenues or something like that.


ANALYZE EACH ANSWER:

Which one of the following, if true, most seriously undermines the argument’s stated conclusion?

(A) The new post office will have to be built in accordance with a demanding new citywide building code.
Wrong. Out of scope. Nothing about building code.

(B) If the new post office is built on the outskirts of town, it will require a parking lot, but if sited near the present post office it will not.
Correct. Total Construction cost = Acquiring site cost + Building parking lot cost.
The cost to build a parking may affect the conclusion negatively. Thus, B is correct answer.

(C) If the new post office is built on the outskirts of town, current city bus routes will have to be expanded to provide access.
Wrong. Expanding city bus routes is NOT the cost of building a post office.

(D) If the new post office is built on the outskirts of town, residents will make decreased use of post office boxes, with the result that mail carriers will have to deliver more mail to homes.
Wrong. SHELL GAME. D talks about the future revenue of the post office. But the future revenue does not affect anything to total building cost. Decreasing revenue and cheap building cost are two different categories. Keep in mind, The conclusion does not ask you to evaluate the plan of building a post office benefit or not. It only addresses the INITIAL cost – COST of building a post office. That's it.

(E) If the new post office is built near the center of town, disruptions to city traffic would have to be minimized by taking such steps as doing some construction work in stages at night and on weekends.
Wrong. Out of scope. Nothing about disruption to city traffic.

Hope it helps.
Manager
Manager
Joined: 16 Jul 2023
Posts: 128
Own Kudos [?]: 13 [0]
Given Kudos: 281
Location: India
GPA: 3.46
Send PM
Re: Brownleas post office must be replaced with a larger one. The present [#permalink]
Bunuel wrote:
RaviChandra wrote:
Brownlea’s post office must be replaced with a larger one. The present one cannot be expanded. Land near the present location in the center of town is more expensive than land on the outskirts of town. Since the cost of acquiring a site is a significant part of the total construction cost, the post office clearly could be built more cheaply on the outskirts of town.

Which one of the following, if true, most seriously undermines the argument’s stated conclusion?


(A) The new post office will have to be built in accordance with a demanding new citywide building code.

(B) If the new post office is built on the outskirts of town, it will require a parking lot, but if sited near the present post office it will not.

(C) If the new post office is built on the outskirts of town, current city bus routes will have to be expanded to provide access.

(D) If the new post office is built on the outskirts of town, residents will make decreased use of post office boxes, with the result that mail carriers will have to deliver more mail to homes.

(E) If the new post office is built near the center of town, disruptions to city traffic would have to be minimized by taking such steps as doing some construction work in stages at night and on weekends.


OFFICIAL EXPLANATION



OA : (B)

OE : Curve ball: The argument starts off with one argument (sentences one + two: We must build a new post office because the old one can’t be expanded), but moves past that first decision to another, more problematic one: Where can the building be built cheaper? The conclusion that the outskirts site is “clearly” cheaper is the one we seek to undermine. Sure, land is cheaper on the outskirts than in town. But what (you may well have asked yourself) about other factors—other issues that might impact the cost? If (B) is true, then there’s a major cost of the outskirts site, building a parking lot, that wouldn’t be incurred in town. Notice that (B) doesn’t prove that the center of town would be the cheaper site. It simply casts doubt on the certainty of the author’s judgment that building on the outskirts would be cheaper. Given (B), we’re left with a need for more evidence in order to choose between the center location with its land costs, and the outskirts site with its parking lot costs. And the very need for more evidence proves that the argument has been undermined.


(A) Even assuming that that new building code would affect the cost of construction at all, let alone in a major way, this code inferably applies to both sites and hence cannot affect the site comparison.

(C), (D) Running extra buses to the outskirts site (C), or delivering more mail (D), would likely cost the city a few bucks. But they’re not construction bucks—they’re not part of the
building costs—and the argument is specifically about which site would be cheaper to build. Therefore, neither choice has any effect on the logic whatsoever.

(E), if anything, strengthens the logic. Adding yet another cost (however much it may cost to work nights and weekends) to the city center site makes the outskirts site seem that much cheaper in comparison.

• Often, when an LSAT question asks you to weaken logic, two or three of the choices will be outside the scope while one will act as a strengthener. Not a hard and fast
rule, but a useful pattern to know of and watch for.

• Principle: Forcing the author to come up with more evidence to support her conclusion is proof that that conclusion has been undermined. (B) illustrates this quite well; see explanation above.

Why are we assuming that we need to BUILD NEW parking lot? What if it already exists or the parking lot is constructed by private firm who will let post office rent some slot making it a variable cost?

Posted from my mobile device
GMAT Club Bot
Re: Brownleas post office must be replaced with a larger one. The present [#permalink]
Moderators:
GMAT Club Verbal Expert
6920 posts
GMAT Club Verbal Expert
238 posts
CR Forum Moderator
832 posts

Powered by phpBB © phpBB Group | Emoji artwork provided by EmojiOne